LSAT and Law School Admissions Forum

Get expert LSAT preparation and law school admissions advice from PowerScore Test Preparation.

User avatar
 Dave Killoran
PowerScore Staff
  • PowerScore Staff
  • Posts: 5853
  • Joined: Mar 25, 2011
|
#46081
Complete Question Explanation
(The complete setup for this game can be found here: lsat/viewtopic.php?t=10395)

The correct answer choice is (B)

Like question #18, the wording of the condition in the question stem is quite tricky, and you must carefully consider the meaning of the statement. The phrase, “I serves on every subcommittee on which M serves,” means that when M serves on a subcommittee, then I also serves on that subcommittee. This relationship is diagrammed as:

  • ..... ..... ..... ..... ..... M ..... :arrow: ..... I

This condition means that M cannot serve on all three subcommittees (if M did serve on all three, then I would have to serve on all three also, a violation of the numerical distribution). Consequently, we can deduce that P must serve on all three subcommittees. We can also infer that M will not serve on two subcommittees (again, I would serve on two as well, a violation of the distribution) and that M serves on only one subcommittee:
J05_Game_#4_#19_diagram 1.png
Because F and G are in a not-block, we know that F and G must serve on different subcommittees:
J05_Game_#4_#19_diagram 2.png
The only remaining uncertainty is the placement of H, and whether H or I is doubled:
J05_Game_#4_#19_diagram 3.png
Answer choice (A): As discussed in the question analysis, M cannot serve on all three subcommittees, and this answer choice is incorrect.

Answer choice (B): This is the correct answer because I could be the member who serves on two subcommittees.

Answer choice (C): This answer choice is incorrect because I cannot serve on all three subcommittees.

Answer choice (D): This answer choice is incorrect because, as shown above, F and M cannot serve on the same subcommittee.

Answer choice (E): This answer choice is incorrect because, as shown above, G and M cannot serve on the same subcommittee.
You do not have the required permissions to view the files attached to this post.

Get the most out of your LSAT Prep Plus subscription.

Analyze and track your performance with our Testing and Analytics Package.